Q: Is 1/2n Convergent or Divergent? Help Appreciated

  • Thread starter Thread starter srfriggen
  • Start date Start date
  • Tags Tags
    Convergence
Click For Summary
The series 1/2n is being evaluated for convergence or divergence. The limit as n approaches infinity is zero, but this alone does not determine convergence. The harmonic series is known to be divergent, and since 1/n is greater than 1/2n, a comparison can be made. It is established that if a divergent series is multiplied by a nonzero constant, it remains divergent. Thus, the series 1/2n is concluded to be divergent.
srfriggen
Messages
304
Reaction score
7
Q: Is the series 1/2n convergent or divergent? If convergent find it's sum.

I see that the limit as n goes to infinity is zero but that does not prove C or D. It seems like it would be divergent but why? How can that be shown?

Any direction/help would be greatly appreciated.
 
Physics news on Phys.org
Try an integral test. Do you know that one?
 
Yes I do but I shouldn't lol (we haven't gotten to that in class). We have covered the harmonic series and shown it is D.

I also notice I can't do a comparison to 1/n since 1/n > 1/2n.
Would the argument 1/2 * 1/n = 1/2 x divergent = D make sense?
 
srfriggen said:
Yes I do but I shouldn't lol (we haven't gotten to that in class). We have covered the harmonic series and shown it is D.

I also notice I can't do a comparison to 1/n since 1/n > 1/2n.
Would the argument 1/2 * 1/n = 1/2 x divergent = D make sense?

Sure. If a series is divergent, then any nonzero constant times the series is also divergent. That's true. But do you have a theorem or something to justify it rather than just writing divergent*(1/2)=divergent? That just seems a little sloppy.
 
Yeah I know. I'm typing on this forum from my phone so it's tough to be elegant lol. The question only asks C or D but I kinda wanted to know for my own purposes but thanks for the help in understanding it. :)
 
Question: A clock's minute hand has length 4 and its hour hand has length 3. What is the distance between the tips at the moment when it is increasing most rapidly?(Putnam Exam Question) Answer: Making assumption that both the hands moves at constant angular velocities, the answer is ## \sqrt{7} .## But don't you think this assumption is somewhat doubtful and wrong?

Similar threads

  • · Replies 2 ·
Replies
2
Views
2K
  • · Replies 3 ·
Replies
3
Views
1K
  • · Replies 2 ·
Replies
2
Views
2K
  • · Replies 5 ·
Replies
5
Views
2K
  • · Replies 1 ·
Replies
1
Views
2K
  • · Replies 16 ·
Replies
16
Views
2K
  • · Replies 4 ·
Replies
4
Views
2K
  • · Replies 14 ·
Replies
14
Views
2K
Replies
14
Views
2K
  • · Replies 2 ·
Replies
2
Views
2K